-1
$\begingroup$

One more motivated by recent questions of Zhi-Wei Sun.

Let $S_n$ be the group of permutations of $\{1,2,\ldots, n\}$.

Is it true that, for every $n \ge 8$, there is at least one even permutation $\pi \in S_n$ and at least one odd permutation $\tau \in S_n$ with $$\sum_{k=1}^n \frac{1}{k \, \pi(k)} = \sum_{k=1}^n \frac{1}{k \, \tau(k)} = 1?$$

One case for each $n$ is not hard; I made it a math.stackexchange question that was successfully answered in 12 minutes. Hopefully the other case is more interesting.


Clarification: As per the MSE question referenced above and Zhi-Wei's comment, the $n$-cycle $(1,2, \dots, n) \in S_n$ satisfies the sum condition. An $n$-cycle is an odd permutation for even $n$ and an even permutation for odd $n$.

Here are the remaining parts of the conjecture.

a. For $n$ even and $n \ge 8$, there is an even $\pi \in S_n$ satisfying $\sum_{k=1}^n \frac{1}{k \, \pi(k)} = 1$.

b. For $n$ odd and $n \ge 9$, there is an odd $\tau \in S_n$ satisfying $\sum_{k=1}^n \frac{1}{k \, \tau(k)} = 1$.


Here are the numbers of even and odd permutations satisfying the sum condition for small $n$.

\begin{array}{c|rr} n\backslash \text{sgn} & +1 & -1 \\ \hline 1 & 1 & 0 \\ 2 & 0 & 1 \\ 3 & 2 & 0 \\ 4 & 0 & 2 \\ 5 & 4 & 0 \\ 6 & 0 & 2 \\ 7 & 4 & 0\\ 8 & 6 & 4\\ 9 & 12 & 24\\ 10 & 90 & 88 \end{array}

One of the first ``interesting'' permutations is the even permutation (in cycle notation) $(1,2,5,8,7,6)(3,4) \in S_8$ which gives \begin{align*} \frac{1}{1\cdot2} + \frac{1}{2\cdot5}+ \frac{1}{3\cdot4}+ \frac{1}{4\cdot3}+ \frac{1}{5\cdot8}+ \frac{1}{6\cdot1}+ \frac{1}{7\cdot6}+ \frac{1}{8\cdot7}\\ = \frac{1}{2} + \frac{1}{10}+ \frac{1}{12}+ \frac{1}{12}+ \frac{1}{40}+ \frac{1}{6}+ \frac{1}{42}+ \frac{1}{56}=1. \end{align*} Not coincidentally, $n=8$ is the smallest value for which there are non-$n$-cycle permutations that satisfy the sum condition.

$\endgroup$

1 Answer 1

0
$\begingroup$

Clearly, \begin{align}&\frac1{1\cdot2}+\frac1{2\cdot3}+\ldots+\frac1{(n-1)n}+\frac1{n\cdot1} \\=& 1-\frac12+\frac12-\frac13+\ldots+\frac1{n-1}-\frac1n+\frac1n=1. \end{align} If $n=2k+1$, then we also have \begin{align}&\frac1{1\cdot2}+\frac1{2\cdot3}+\ldots+\frac1{k(2k+1)}+\frac1{(2k+1)(k+1)} \\&+\frac1{(k+1)(k+2)}+\cdots+\frac1{(2k-1)2k}+\frac1{2k\cdot1} \\=&1.\end{align} See also my related conjectures available from http://oeis.org/A322069 and http://oeis.org/A322070.

$\endgroup$
1
  • $\begingroup$ @ Zhi-Wei Your first permutation, $\pi = (2,3,\ldots, n, 1)$, is the ``not hard'' $n$-cycle solution from the math.stackexchange problem. When $n$ is even, $\pi$ is odd and vice versa. Your second permutation is a different $n$-cycle for $n$ odd, giving another even permutation. The remaining parts of the question ask for an even permutation of $S_{2k}$ (with $k \ge 4$) and an odd permutation of $S_{2k+1}$ (also with $k \ge 4)$ that satisfy the sum condition. $\endgroup$ Nov 26, 2018 at 4:04

Your Answer

By clicking “Post Your Answer”, you agree to our terms of service and acknowledge you have read our privacy policy.

Not the answer you're looking for? Browse other questions tagged or ask your own question.